Proving the range of a piecewise function

In summary, the conversation discusses how to prove that the range of the function f, defined as f(n) = {n-1 if n is even, n+5 if n is odd}, is equal to the set of all integers ℤ. The writer is unsure of how to proceed with the proof, but mentions that they must show that ℤ is a subset of the range and vice versa. They also mention that the function creates two sets of points, one along the line y = x - 1 and the other along the line y = x + 5. The conversation is then clarified, stating that the range is actually ℤ, not 2 as previously stated. The writer then continues to ask for help with the
  • #1
ver_mathstats
260
21

Homework Statement


Allow f:ℤ→ℤ be defined by, for all n∈ℤ

f(n) = {n-1 if n is even, n+5 if n is odd

Prove that ran(f) = ℤ

Homework Equations

The Attempt at a Solution


I am unsure of how exactly to prove this due to the fact now I am working with a piecewise function.
Here is what I have so far:
We must demonstrate that ran(f) ⊆ ℤ and that ℤ ⊆ ran(f).
I know that when n is odd, f(n) is even and that when n is even, f(n) is odd. And now I know I must show that a certain integer is in the range. And I know that if m is odd, m + 1 will be even and that when m is even, m - 5 will be odd, now I do not know where to go from here.
Thank you.
 
Last edited:
Physics news on Phys.org
  • #2
ver_mathstats said:

Homework Statement


Allow f:ℤ→ℤ be defined by, for all n∈ℤ

f(n) = {n-1 if n is even, n+5 if n is odd

Prove that ran(f) = 2
I don't see how this can be true at all. If you plot a graph of this function, you get two sets of points, one along the line y = x - 1, and the other along the line y = x + 5.
ver_mathstats said:

Homework Equations

The Attempt at a Solution


I am unsure of how exactly to prove this due to the fact now I am working with a piecewise function.
Here is what I have so far:
We must demonstrate that ran(f) ⊆ 2 and that 2 ⊆ ran(f).
Firstly, do I have to split this into cases? I think I must, is this right?
I started working with n-1, if n is even and I got let y ∈ ran(f). By definition of range we can fix x ∈ ℝ such that y = n-1, however since n must be even must we define what even is and substitute it into n? Resulting in y=2k-1. I am unsure of where to go from here if this is correct so far.

Thank you.
 
  • #3
Mark44 said:
I don't see how this can be true at all. If you plot a graph of this function, you get two sets of points, one along the line y = x - 1, and the other along the line y = x + 5.
There was typo, but it was clarified it was supposed to be ran(f) = ℤ. My apologies, I will fix it right now.
 
  • #4
ver_mathstats said:
There was typo, but it was clarified it was supposed to be ran(f) = ℤ. My apologies, I will fix it right now.
That makes a lot more sense. Do you see why every integer in ##\mathbb Z## is the image of some other number in this set?
 

What is a piecewise function?

A piecewise function is a mathematical function that is defined by different formulas or rules for different parts of its domain. This allows the function to have different behaviors in different intervals or regions.

Why is it important to prove the range of a piecewise function?

Proving the range of a piecewise function is important because it helps us understand the behavior of the function and its output values. It also allows us to identify any gaps or missing values in the range, which can affect the overall interpretation of the function.

How do you prove the range of a piecewise function?

The range of a piecewise function can be proved by analyzing the domain and range of each individual piece of the function and then combining them to determine the overall range. This can be done by graphing the function or by using algebraic methods such as substitution and solving equations.

What are some common challenges when proving the range of a piecewise function?

One common challenge when proving the range of a piecewise function is identifying the correct domain and range for each piece of the function. Another challenge is determining how to combine the individual ranges to find the overall range, especially if there are gaps or overlaps between the pieces.

Are there any tips for proving the range of a piecewise function?

One helpful tip for proving the range of a piecewise function is to carefully analyze the behavior of the function at the points where the pieces connect. This can help determine if the function is continuous or discontinuous at those points, which can affect the overall range. It can also be helpful to graph the function to visualize its behavior and range.

Similar threads

  • Calculus and Beyond Homework Help
Replies
16
Views
1K
  • Calculus and Beyond Homework Help
Replies
9
Views
804
  • Calculus and Beyond Homework Help
Replies
2
Views
1K
Replies
5
Views
2K
  • Calculus and Beyond Homework Help
Replies
6
Views
381
  • Calculus and Beyond Homework Help
Replies
3
Views
544
  • Calculus and Beyond Homework Help
Replies
3
Views
410
  • Calculus and Beyond Homework Help
Replies
1
Views
501
  • Calculus and Beyond Homework Help
Replies
5
Views
1K
  • Calculus and Beyond Homework Help
Replies
3
Views
1K
Back
Top